Sie sind auf Seite 1von 22

1.

Brunhilda: Economists have predicted that our generation will be the first that cannot
confidently look forward to having a better standard of living than that enjoyed by our parents.

Siegfried: That's simply untrue. My father's standard of living is nowhere near as high as his
parents' was, and my own standard of living is already higher than that of my parents.

Which of the following best describes the error of reasoning contained in Siegfried's argument
above?
A. It relies upon an unreasonable appeal to authority.
B. It assumes the truth of what it sets out to prove.
C. It offers an example that is not inconsistent with Brunhilda's argument.
D. It is based on an unproven speculation about future events.
E. It uses evidence of a correlation to argue the existence of a causal relationship.

OA: A

This question uses one of the most common fallacies "Appeal to authority"

The fallacy form is:


A is an authority/expert on subject X
A makes claim Y about subject X
Therefore, Y is true.

APPLY TO THIS QUESTION:

I will paraphrase a bit to make the stimulus follow the fallacy above:

* Brunhilda: Economists have predicted that our generation will be the first that cannot confidently
look forward to having a better standard of living than that enjoyed by our parents.

* Siegfried: [From my experiences] My father's standard of living is nowhere near as high as his
parents' was, and my own standard of living is already higher than that of my parents. So your point is
simply untrue

Siegfried assumes his own experience is comprehensive enough ==> He is an expert /authority of this
matter ==> His conclusion must be true.

This error is very common. When you hear some one says:
The scientists say that.....
I have a book that says......
I read a newspaper that says......
I saw on TV.........
From my own experience, I think.......

Immediately, you should think about the fallacy "appeal to authority". You should ask:
The scientists have expertise on this matter ?
The book is believable ?
The information on newspaper is true ?
The TV channel is biased or not?
Your own experience is comprehensive enough?
etc.......
ANALYZE EACH ANSWER

A. It relies upon an unreasonable appeal to authority.


Correct. As stated above.

B. It assumes the truth of what it sets out to prove.


Wrong. The truth of what Siegfried said is actually true (from his family experience), not out of prove
as B says.

C. It offers an example that is not inconsistent with Brunhilda's argument.


Wrong. Shell Game. The language used in C may make you think C is correct, but it's not. Actually,
Siegfried opposed what Brunhilda mentioned because he said "this is simply untrue".

D. It is based on an unproven speculation about future events.


Wrong. Siegfried did not base his argument on future events, actually he mentioned something in the
past (his family experience)

E. It uses evidence of a correlation to argue the existence of a causal relationship.


Wrong. Siegfried did not oppose to any causal relationship here.

2. The Daisygen Company genetically engineers flowers which are then sent to retailers in
locations around the world. By splicing a rose with a species of onion they managed to create a
flower that looks like the classic, and still very popular, red rose, but that takes much longer to
wilt. A longer expiration period presents an important advantage to international flower
importers and to individual consumers. Clearly, sales of the Daisygen rose will outnumber those
of the original rose species.

Which of the following is an assumption underlying the conclusion?

A. The genes which were spliced into the rose are not also responsible for the onion's tearing
effect.
B. Consumers are interested in a flower that will last longer.
C. Plastic flowers have not affected the import and export of natural flowers.
D. Although extremely popular, the original rose was not the highest selling flower species of
all.
E. A longer expiration period will allow importers more time for delivery of the Daisygen roses.

OA: A
This question uses a common fallacy "appeal to novelty"

The form is:


X is newer than Y
--OR--
X has new characteristics that are better than those of Y.
Therefore, X is correct, better, or preferable.

The conclusion is true only if:


(1) X is at least as good as Y (retain original advantages)
--OR--
(2) X does not create new weaknesses that Y does not have.
ANALYZE THE STIMULUS:

Fact: By splicing a rose with a species of onion they managed to create a flower that looks like the
classic, and still very popular, red rose, but that takes much longer to wilt.
Fact: A longer expiration period presents an important advantage to international flower importers
and to individual consumers.
Conclusion: Sales of the Daisygen rose will outnumber those of the original rose species.

Apply the logic above: the new rose is preferable only if:
(1) it remains advantages of the original rose
--OR--
(2) the new rose does not “create” new weaknesses that the original rose does not have.

ANALYZE EACH ANSWER:

a)The genes which were spliced into the rose are not also responsible for the onion's tearing effect.
Correct. To conclude sales of new roses will outnumber those of the original roses, the new roses
MUST NOT create disadvantages that the original roses do not have.

b)Consumers are interested in a flower that will last longer.


Wrong. TEMPTING. You will see this kind of trap over and over again in real GMAT test. B is wrong
because B is incomplete assumption.The complete B is: Consumers are interested in a flower
that does not have new weaknesses and will last longer (new advantage). Thus, B exposes
weakness for criticism and is incorrect assumption.

c)Plastic flowers have not affected the import and export of natural flowers.
Wrong. Out of scope. Nothing about “plastic flowers”.

d)Although extremely popular, the original rose was not the highest selling flower species of all.
Wrong. “highest selling” is out of scope.

e)A longer expiration period will allow importers more time for delivery of the Daisygen roses.
Wrong. “more time for delivery” is the fact, not the assumption.

3. Two types of earthworm, one black and one red-brown, inhabit the woods near the town of
Millerton. Because the red-brown worm's coloring affords it better camouflage from predatory
birds, its population in 1980 was approximately five times that of the black worm. In 1990, a
factory was built in Millerton and emissions from the factory blackened much of the woods.
The population of black earthworms is now almost equal to that of the red-brown earthworm, a
result, say local ecologists, solely stemming from the blackening of the woods.

Which of the following, if true, would most strengthen the conclusion of the local ecologists?

A]The number of red-brown earthworms in the Millerton woods has steadily dropped since the
factory began operations.
B]The birds that prey on earthworms prefer black worms to red-brown worms.
C]Climate conditions since 1990 have been more favorable to the survival of the red-brown
worm than to the black worm.
D] The average life span of the earthworms has remained the same since the factory began
operations.
E]Since the factory took steps to reduce emissions six months ago, there has been a slight
increase in the earthworm population.
This is an example of the combination of CAUSES & EFFECTS and COMPARISON.

MAIN IDEA:

Before event A, X is less than Y.


After event A, X and Y are equal
Conclusion: Event A causes X increase.

Assumption: Y does not decrease itself or there's NO factors make Y decrease.

Negation to confirm: If Y decreases itself ==> cannot say Event A causes X increase (because X
is the same, only Y decreases to make two variables are equal).

CONSIDER EACH ANSWER:

(A) The number of red-brown earthworms in the Millerton woods has steadily dropped since the
factory began operations.
Wrong. Actually A weakens the conclusionn not strengthen because it says red-brown
earthworms decreased by itself.

(B) The birds that prey on earthworms prefer black worms to red-brown worms.
Wrong.. Shell game. Birds prefer black worms ==> why birds prefer black worms, because birds
can see black worms ==> if they cannot see black worms, how can they prefer them? ==> we
can infer that the blackening of the woods does not play any role to help black worms.

(C) Climate conditions since 1990 have been more favorable to the survival of the red-brown
worm than to the black worm.
Correct. C definitely says the number of red worms does not decrease, even red worms
increased their population more than in 1980 ==> However, the number of black worms equals
to that of red worms ==> the the blackening of the woods must play an important role to help
black worms.

(D) The average life span of the earthworms has remained the same since the factory began
operations.
Wrong.. D only says the AVERAGE life span of earthworms unchanged. Let analyze an example.

Before: The number of Red earthworms is five times that of Black earthworms. (100 vs 20)
There are 20 Black earthworms (BW), their life span is 11 days
There are 100 Red earthworms (RW), their life span is 5 days
Average life span = [20*11 + 100*5] / 120 = 6 days

After. The number of Red earthworms equals that of Black earthworms. (20 vs 20)
There are 20 BW, their life span is 11 days
There are 20 RW, their life span is 1 day (Let say, Emission from factory affected only RW,
leading to RW reduced both its number and its life span. Blackening did not affect BW at all)
Average life span = [20*11 + 20*1] /40 = 6 days

(E) Since the factory took steps to reduce emissions six months ago, there has been a slight
increase in the earthworm population.
Wrong.. Apposite answer. Factory reduces emission ==> woods are less blacken ==> But the
number of earthworm increase ==> the blackening of the woods does not play any role.
4. For each of the past two years, major department stores have reported a nearly 50% increase
in their sales of men's clothing manufactured by Zachary, Inc., a result that is all the more
surprising because the sales of most other brands of men's clothing have been depressed over
the same period. Nevertheless, even Zachary, Inc. does not appear to have emerged unscathed
from the overall trend: despite the aforementioned increase, Zachary, Inc. has reported a
slight decline in overall sales in each of the past two years.

Which of the following, if true, most helps to explain the surprising result above?

A) The sales of clothing at Zachary, Inc.'s boutique stores, which, unlike department stores, are
owned and operated by the company itself, have held steady over the last two years.

B) Two years ago, Zachary, Inc. began an ambitious new advertising campaign; in each of the
last two years, the company's advertising department has overspent its planned budget by
almost half.

C) Zachary, Inc. is renowned for the quality of its fabrics, and sells large quantities of fabric to
other manufacturers of men's clothing.

D) Zachary, Inc. formerly manufactured leather accessories and women's clothing in addition to
men's clothing, but, for the past three years, the company has produced only men's clothing.

E) In the last two years, the percentage of Zachary, Inc.'s clothing sold by department stores at
discounted prices has been unusually high, a result that analysts have blamed on the sluggish
economy.

GENERAL METHOD:

For Resolve the Paradox questions, we must select an answer that can explain both sides of the
situation, not only one side. That's the KEY.
Before solving this question, please note that there are two types of answers ALWAYS are
incorrect:
1. Explain only one side of the paradox.
2. Just explain the similarities and differences, e.g. if the stimulus contains a paradox where
two items are similar, then an answer choice explains a difference between the two cannot be
correct.

APPLY:

The logic here is: men clothing manufactured by Zachary increased 50%, but overall sales of
Zachary slightly decreased. You see the gap?OVERALL Sales do not mean ONLY men clothing. If
you see an answer that shows Zachary men clothing sales increased, but sales of Zachary's
products OTHER THAN men clothing decreased. That would be the answer. Because OVERALL
SALES = MEN CLOTHING + OTHER PRODUCTS. Only men clothing sales increased is not
enough to conclude Overall sales increased.

CONSIDER EACH ANSWERS:

A) The sales of clothing at Zachary, Inc.'s boutique stores, which, unlike department stores, are
owned and operated by the company itself, have held steady over the last two years. - WRONG
- Out of scope

B) Two years ago, Zachary, Inc. began an ambitious new advertising campaign; in each of the
last two years, the company's advertising department has overspent its planned budget by
almost half. - WRONG - Out of scope

C) Zachary, Inc. is renowned for the quality of its fabrics, and sells large quantities of fabric to
other manufacturers of men's clothing. - CORRECT - Because Zachary's fabric is famous, so
many people want to buy men clothing manufactured by the company --> Men clothing sales
increased. Besides men clothing, Zachary also sells fabrics to other manufactures. However,
other manufactures sales decreased because of weak economy (mentioned in the stimulus),
making Zachary's overall sales slightly decreased. Because they are also Zachary's customers. This
is the answer!

D) Zachary, Inc. formerly manufactured leather accessories and women's clothing in addition to
men's clothing, but, for the past three years, the company has produced only men's clothing. -
WRONG - Out of scope.

E) In the last two years, the percentage of Zachary, Inc.'s clothing sold by department stores at
discounted prices has been unusually high, a result that analysts have blamed on the sluggish
economy. - WRONG - Please review my note above, the answer that explains only one side is
ALWAYS incorrect. E only explains why men clothing sales of Zachary increased, but doesn't
explain why Zachary's overall sales decreased. That cannot be the answer.

5. Pretzels can cause cavities. Interestingly, the longer that a pretzel remains in contact with the
teeth when it is being eaten, the greater the likelihood that a cavity will result. What is true of
pretzels in this regard is also true of caramels. Therefore, since caramels dissolve more quickly
in the mouth than pretzels do, eating a caramel is less likely to result in a cavity than eating a
pretzel is.
The reasoning in the argument is vulnerable to criticism on the grounds that the argument

(A) treats a correlation that holds within individual categories as thereby holding across
categories as well
(B) relies on the ambiguous use of a key term
(C) makes a general claim based on particular examples that do not adequately represent the
respective classes that they are each intended to represent
(D) mistakes the cause of a particular phenomenon for the effect of that phenomenon
(E) is based on premises that cannot all be true

GENERAL METHOD:

This question is Flaw in the Reasoning question that is considered the most difficult question in
GMAT. Flaw questions appear infrequently in GMAT. If you see this kind of question, that would
be a signal you are doing very well on the GMAT.

The technique used in Flaw question is Prephrasing and Fact test. Keep in mind that answers
provide "new information" always be considered wrong.

APPLY:

Now, I will prephrase the stimulus as follows:


Why eating Pretzel makes cavity? Because Pretzel contacts with teeth. There must be a
correlation between Pretzel and teeth. KEY
Why eating Caramel makes cavity? Because Caramel contacts with teeth. There must be a
correlation between Caramel and teeth. KEY
If No, there's no cavity
The longer the correlations remains, the greater the likelihood that a cavity will result.

Author concludes: the correlation between Caramel & teeth lasts shorter than that between
Pretzel and teeth >>>> the less likelihood that a cavity will result.

I bet you see THE GAP. The correlation between Caramel & teeth is DIFFERENT from the
correlation between Pretzel & teeth. We cannot say "this is true for X also true for Y". This is
wrong assumption.

What A says: "treats a correlation that holds within individual categories as thereby holding
across categories as well". KEY WORD: "holds within & holding across"

It means: Author treats a correlation between Caramel & teeth and a correlation between
Pretzel & teeth the same, and thereby makes a conclusion.

A clearly shows the flaw in author's reasoning.

6. Of all the times for the U.S. to be discouraging domestic production of oil and natural gas, right
now might be the worst. Libya's descent into chaos is fueling a rapid rise in oil prices, and
unrest in other oil-producing countries in the Middle East and North Africa has led some
analysits to predict unprecedented oil-price spikes may be looming.
If the statements above are true, which of the following inference can be most properly drawn?
A) The oil prices at present are the highest of what they might have been in the last few
decades.
B) Libya is a major oil producing country.
C) If US allows oil domestic oil production, it may be able to avoid any further price increase.
D) The oil price increases that are predicted by analysts may be larger than those in the recent
past.
E) Unrest in oil producing countries in Middle East has impacted overall oil production.

ANALYZE THE STIMULUS:

Factor #1: Libya's descent into chaos is fueling a rapid rise in oil prices.
Factor #2: unrest in other oil-producing countries in the Middle East and North Africa
Sub conclusion: unprecedented oil-price spikes may be looming.
Main conclusion: Of all the times for the U.S. to be discouraging domestic production of oil
and natural gas, right now might be the worst.

KEY WORD: unprecedented oil-price spikes.

ANALYZE EACH OPTION:

If the statements above are true, which of the following inference can be most properly drawn?

A) The oil prices at present are the highest of what they might have been in the last few
decades.
Wrong. Nothing about “oil prices at present are the highest”. Probably, the oil prices at
present are high, but we do not have enough info to conclude they are the highest.

B) Libya is a major oil producing country.


Wrong. SHELL GAME.
- “oil producing” differs from “oil exporting”. Libya may be a major oil exporting country. It
may export a big amount of “rude” oil. So its chaos will have a big impact on oil prices. But,
the premises do not say “Libya is a major producing country”.

C) If US allows oil domestic oil production, it may be able to avoid any further price increase.
Wrong. SHELL GAME. It’s wrong because:
- If US allows oil domestic production, it may be able to avoid further price increase in the U.S
only. But, C say US can avoid price increase in general. C means U.S only or worldwide?
- C say the US can avoid any further price increase. KEY WORD is “any”. Is it true that the US
can avoid any further price increase? The US can avoid only price increase to some extent, not
any further price increase.

D) The oil price increases that are predicted by analysts may be larger than those in the recent
past.
Correct. The stimulus says: “unprecedented oil-price spikes”. “unprecedented” means “have
never happened before”. D is the most properly inference.

E) Unrest in oil producing countries in Middle East has impacted overall oil production.
Wrong. It’s true that unrest in oil producing countries in ME will impact overall oil production.
But the unrest did impact or will impact (the time of the impact)? We don’t know. Hence, E is
wrong.

7. Now problems require new solutions. And new problems arise with new populations and new
technologies. The solutions of these problems require new institutions as well as new political,
economic, and social mechanisms. Yet institutions and political and economic arrangements
grow slowly, new institutions should be given every chance of success.
The writer of the above makes which of the following assumptions:
(A) New institutions are needed because old institutions are inefficient.
(B) New institutions are created in order to solve existing problems
(C) As old institutions are phased out, new ones take their place.
(D) If there were no growth, old institutions would die more slowly.
(E) Socio-technological change requires new forms of institutional arrangement.

ANALYZE THE STIMULUS:

Fact 1: Now problems require new solutions.


Fact 2: The solutions of these problems require
New institutions - factor #1
--AND--
New political, economic, and social mechanisms - factor #2.
Fact 3: But factor #2: institutions and political and economic arrangements grow slowly.
Conclusion: Factor #1: new institutions should be given EVERY CHANCE of success.

KEY is to how translate correctly the conclusion. The fact says: new problems need new
solutions, which require both factor (1) and factor (2). But because factor (2) grows slowly,
so the key to have new solutions created is “give every chances of success” for (1): new
institutions.

On the other hand, “every chances of success” means it doesn’t matter how many time new
institutions are failed. If they fail, the new ones should be created.

The writer of the above makes which of the following assumptions:

ANALYZE EACH ANSWER:

(A) New institutions are needed because old institutions are inefficient.
Wrong. Out of scope. The main point is giving new institution EVERY CHANCES. Thus, it doesn’t
matter whether the old institution are ineffective.

(B) New institutions are created in order to solve existing problems


Wrong. TEMPTING but wrong. New institution solve ONLY existing problem? Nope, The stimulus
clearly mentioned the existing of “NEW” problems (new problems arise ...). Why B narrows the
scope to “existing problems”? B is incorrect because it does not cover enough.

(C) As old institutions are phased out, new ones take their place.
Correct. C restates exactly what the conclusion wants to convey. “give every chances of
success” means it doesn’t matter how many time new institutions are failed. If they fail, the
new ones should be created.

(D) If there were no growth, old institutions would die more slowly.
Wrong. Out of scope. Same as B. We do not discuss the old institutions. The main point is the
creating of new institutions. It does not matter the old institutions exist or not. They are in
different categories. Thus, D is wrong.

(E) Socio-technological change requires new forms of institutional arrangement.


Wrong. The stimulus says we could ignore factor 2: the institutions and political and economic
arrangements, because they grow slowly. Thus, the argument just focus on factor 1: the
creating of new institutions which should be given EVERY CHANCES of success.

8. The snack food market has recently seen an explosion in the production and sales of "100-
Calorie packs," individually wrapped portions of snack foods sized to provide exactly 100
calories per portion. These packs cost substantially more per ounce-they sell for nearly same
price as traditionally sized portions, which are typically two to three times their size - but
consumers have been purchasing them with greater and greater frequency. One possible
explanation is convenience: Consumers may be relatively unconcerned with their subjective
impression of the small packs, and willing to pay more to avoid having to measure out their
portions. Another possible explanation is that 100- calorie pack, at least in the case of snacks
for which it has sold well, represents the smallest portion that still looks and feels "substantial"
enough to appeal to dieters who lack the self control to limit their consumption of snacks from
larger packages.
Which of the following, if true, would support one of the given explanation and undermine the
other?
A. Consumers are willing to pay exactly the same price for 100-calorie packs sold in vending
machine as for traditionally sized snack portions sold in vending machine.
B. A large number of buyers of 100-calorie packs consume them as light desserts after large
meals that have left them feeling too full for traditionally sized snack food.
C. Although the 100-calorie packs have begun to sell well across a large variety of
demographics, busy young professionals were the first group to purchase them frequently.
D. Because the 100-calorie packs require more packaging per ounce of food than the
traditionally sized portions do, manufacturers must charge more per ounce to make the same
relative profit as on traditionally sized portions.
E. Sales of 100-calorie packs have been uniformly poor at stores where they are displayed
alongside traditionally sized portions, even for the same snacks whose 100-calorie packs are
bestsellers at other stores.

Very tricky question, IMO E is correct.

Fact: 100Calories packs cost more per ounce-than traditionally sized portions.
Fact: Consumers have been purchasing 100Calories pack with greater and greater frequency.

Explanation 1: Consumers are willing to pay more to avoid having to measure out their
portions. ==> convenience is 1st priority.

Explanation 2: 100Calories pack represents the smallest portion that still looks and
feels "substantial" enough to appeal to dieters who lack the self control to limit their
consumption of snacks from larger packages. ==> Because 100C pack is large enough so even if it
stands alone, it still attracts dieters. However, if 100c pack stands beside a LARGER sized portion,
dieters will ignore the smaller one and pick up the bigger one.

A. Consumers are willing to pay exactly the same price for 100-calorie packs sold in vending
machine as for traditionally sized snack portions sold in vending machine.
Wrong. Out of scope.

B. A large number of buyers of 100-calorie packs consume them as light desserts after large
meals that have left them feeling too full for traditionally sized snack food.
Wrong. Out of scope.

C. Although the 100-calorie packs have begun to sell well across a large variety of
demographics, busy young professionals were the first group to purchase them frequently.
Wrong. Good shell game. It maybe fits the first explanation, but It's wrong because "the first
group to purchase" does not mean busy young professionals will keep their habit. They are just
the group of people who always try "NEW stuffs" more frequently than other group of people.

D. Because the 100-calorie packs require more packaging per ounce of food than the
traditionally sized portions do, manufacturers must charge more per ounce to make the same
relative profit as on traditionally sized portions.
Wrong. Out of scope.

E. Sales of 100-calorie packs have been uniformly poor at stores where they are displayed
alongside traditionally sized portions, even for the same snacks whose 100-calorie packs are
bestsellers at other stores.
Correct. Because 100C pack is large enough so even if it stands alone, it still attracts dieters.
==> If 100c pack stands beside LARGER sized portion, dieters will ignore 100c pack. ==> E also
undermines the 2nd explanation because if convenience is priority, customers should ignore the
large sized portions to pick up the smaller sized packs.
9. The percentage of people weighing over 150 pounds is higher in Irvine than any other city.
However, the percentage of people weighing over 180 pounds is higher in Tustin.

If the statements above are true, which of the following must also be true?

A) Irvine has the second highest percentage of people weighing over 180 pounds.

B) Average weight is higher in Tustin than in Irvine

C) Some people in Irvine weigh between 150 and 180 pounds

D) The number of people who weigh more than 150 pounds is greater in Irvine than in Tustin.

E) The percentage of people who weigh over 180 pounds is greater in Tustin than in Irvine.

Be aware of "short" questions. They are always more difficult than the longer ones. If you see a
short question, do NOT skim. The trap may be hidden in just one word.

This question is "ALSO must be true", not "must be true" question. The difference between the
two is the correct answer of the later could be a paraphrase of the stimulus, however, the
correct answer of the former cannot be a paraphrase.

ANALYZE THE STIMULUS:

The percentage of people weighing over 150 pounds is higher in Irvine than any other city.
However, the percentage of people weighing over 180 pounds is higher in Tustin.

If the statements above are true, which of the following must also be true?

ANALYZE EACH ANSWER:

A) Irvine has the second highest percentage of people weighing over 180 pounds.
Wrong. Clearly out of scope. Eliminate right away.

B) Average weight is higher in Tustin than in Irvine


Wrong. Cannot say that. We do NOT have information about people in other categories (< 150
pounds)

C) Some people in Irvine weigh between 150 and 180 pounds


Correct. This is an "ALSO" must be true answer. Because the stimulus say Irvine has some people
weighting more than 150 pounds, and the percentage of people weighting over 180 pounds
higher in Tustin (than in Irvine <-- Because the stimulus clearly is comparing Tustin and Irvine).
==> We are 100% sure that Irvine has some people who weight between 150 and 180 pounds

D) The number of people who weigh more than 150 pounds is greater in Irvine than in Tustin.
Wrong. We have ONLY percentage, we CAN'T say anything about absolute value.
For example:
Irvine has 100 people, 20 people weighting over 150 pounds --> percentage = 20%
Tustin has 1000 people, 100 people weighting over 150 pounds --> percentage = 10%
==> Even though the percentage number of Irvine is larger, the absolute value is smaller.
E) The percentage of people who weigh over 180 pounds is greater in Tustin than in Irvine.
Wrong. E is just a paraphrase of the second sentence. It would be correct if the question was
"must be true". However, the question stem clearly says "which ALSO must be true". Thus, the
answer that only repeats the stimulus is wrong.

Hope it helps.

10. A new medical procedure replaces all three of the tiny bones in the inner ear with a single
piece of ultra-thin fiberglass. The procedure has been found to greatly improve hearing in
people who have experienced damage to these bones, though it is useless to people whose
hearing loss stems from a neurological malfunction. This procedure will benefit a relatively
small percentage of the hearing-impaired population

Which of the following can be concluded from the argument above?

A)It is possible to hear without the use of the three tiny bones in the inner ear.
B)Most hearing loss is due to neurological malfunctioning.
C)More people have impaired hearing because of neurological damage than because of damage
to the tiny bones of the inner ear.
D)Hearing loss due to neurological damage is more severe than hearing loss due to damage to
the tiny bones in the inner ear.
E)The use of fiberglass cannot help people who have lost hearing due to neurological damage.

Interesting question. Key to solve this question is “question stem”. This question is
“INFERENCE” question, not “must be true” question.

A little bit difference between the two.

(1) Must be true


- Fact test / No “new info” accepted
- Correct answers (1) Paraphrasing OR (2) Combination

(2) Inference
- Subcategory of Must be true
- Have to pass “Fact test”
- Wrong answers: Only repeat premises

Thus, any answer that only repeat (paraphrase) premises is WRONG. You should “infer” or
conclude a “main point”, not just repeat what the stimulus says in another way.

ANALYZE THE STIMULUS:

Fact #1: A new medical procedure replaces all three of the tiny bones in the inner ear with a
single piece of ultra-thin fiberglass
Fact #2: . The procedure has been found to greatly improve hearing in people who have
experienced damage to these bones, though it is useless to people whose hearing loss stems
from a neurological malfunction
Conclusion: This procedure will benefit a relatively small percentage of the hearing-impaired
population

ANALYZE EACH ANSWER:


A)It is possible to hear without the use of the three tiny bones in the inner ear.
Correct. The fact #2 says the procedure improves hearing in people who have tiny bones in the
inner ear damaged. The word “improve” clearly indicate that people are still possible to
hear even though the hearing quality is not really good.

B)Most hearing loss is due to neurological malfunctioning.


Wrong. The conclusion says: “small percentage of the hearing-impaired population (who have
tiny bones damaged) will benefit”. But “small” differs from “none”. Thus, we cannot infer that
“most hearing loss is due to neurological malfunctioning”.

C)More people have impaired hearing because of neurological damage than because of damage
to the tiny bones of the inner ear.
Wrong. SHELL GAME. It’s not the correct answer. The reason is:
we cant infer this because the argument ONLY states that :
procedure will benefit a relatively small percentage of the hearing-impaired population.

Let say there are 10 types of hearing impairment(including bone damage one and neurologically
impaired)
let say this procedures cures only those impairments which is caused by BONE DAMAGE.....still
the statement holds that :procedure will benefit a relatively small percentage of the hearing-
impaired population.
hence that option is not repeated in the premise.
(I borrowed this example from Blueseas)

D)Hearing loss due to neurological damage is more severe than hearing loss due to damage to
the tiny bones in the inner ear.
Wrong. Out of scope. Nothing about “severe”.

E)The use of fiberglass cannot help people who have lost hearing due to neurological damage.
Wrong. SHELL GAME. This is TRUE. (see fact #2 which says: it’s useless…..) But it’s not the
correct answer. The question does not ask you to find a “must be true” answer. We must find
an "inference" answer.

TAKE AWAY:
- Always read the question stem carefully. Determine exactly the type of question
- “Inference question” differs from “Must be true”. Any answer that only repeat /paraphrase a
premise is WRONG.

11. Government official: Clearly, censorship exists if we, as citizens, are not allowed to
communicate what we are ready to communicate at our own expense or if other citizens are
not permitted access to our communications at their own expense. Public unwillingness to
provide funds for certain kinds of scientific, scholarly, or artistic activities cannot, therefore,
be described as censorship.
The flawed reasoning in the government official’s argument is most parallel to that in which
one of the following?

(A) All actions that cause unnecessary harm to others are unjust: so if a just action causes harm
to others, that action must be necessary.
(B) Since there is more to good manners than simply using polite forms of address, it is not
possible to say on first meeting a person whether or not that person has good manners.
(C) Acrophobia, usually defined as a morbid fear of heights, can also mean a morbid fear of
sharp objects. Since both fears have the same origin.
(D) There is no doubt that a deed is heroic if the doer risks his or her own life to benefit
another person. Thus an action is not heroic if only thing it endangers is the reputation of the
doer.
(E) Perception of beauty in an object is determined by past and present influences on the mind
of the beholder. Thus on object can be called beautiful, since not everyone will see beauty in
it.

GENERAL THEORIES

This is a parallel reasoning question, but it's more like a flaw of reasoning question, so it’s
important to find the fallacy first. If you can’t realize the flaw of reasoning in the stimulus, it’s
difficult to pick a correct choice.

This question uses a fallacy, namely, Denying the Antecedent.


The form is:
If A, then B
Conclusion: If Not A, then Not B.
Note: The logic is WRONG.

For example:
If I have 800 GMAT score, I will be accepted at Harvard Business School.
Conclusion: If I don’t have 800 GMAT score, I will NOT be accepted at HBS.
It’s wrong. Even though 800 is exceptional score. If one has 800 GMAT, HBS will accept him.
However, 800 GMAT is not the ONLY condition to decide an applicant is accepted or not.

APPLY TO THE QUESTION:

If [as citizens, are not allowed to communicate –OR-- other citizens are not permitted access to
our communications], then censorship exists.
Conclusion: if public is unwillingness to provide funds for XYZ, then censorship does NOT exist.

KEY word is “unwillingness” which means people are ALLOWED to provide funds, but not
willing to do so. I will paraphrase the conclusion like:
Paraphrased version of conclusion: If public is ALLOWED to provide funds for XYZ, then
censorship does NOT exist.

This is an alternate version of “Denying the Antecedent”.


Form used in this question is:
If A is negative (not allowed to...), then B positive (censorship exists).
Conclusion: If A is positive (is allowed to....), then B negative (censorship does not exist)..
This is a wrong logic.

Only D has a parallel reasoning.

Let examine D:

(D) There is no doubt that a deed is heroic if the doer risks his or her own life to benefit
another person. Thus an action is not heroic if only thing it endangers is the reputation of the
doer.
The form of D is:
If doer risks his or her own life, then a deed is heroic.
Thus, if doer does not risk his or her own life (the doer only risks his or her reputation), then an
action is not a heroic.

Clearly, the conclusion in D has a similar fallacy as we discussed above. Hence, D is the answer.

12. Although only 2 percent of drivers on Lalaland’s highways drove sports cars, 25 percent of all
vehicles ticketed for drunk driving in the past 90 days were sports cars. Clearly, sports car
drivers on Lalaland highways are more likely to drive drunk than are drivers of other kinds of
vehicles.

The conclusion drawn above depends on which of the following assumptions?

A)Drivers on Lalaland highways drive drunk more often than do drivers on highways not covered
in the report.
B)Many of the vehicles ticketed for drunk driving were ticketed more than once during the time
period covered by the report.
C)Drivers who are ticketed for drunk driving are more likely to drive drunk regularly than are
drivers who are not ticketed.
D)The number of drivers ticketed for drunk driving was greater than the number of sportscars.
E)Drivers of sports cars are less likely to be ticketed for drunk driving than are drivers of other
kinds of cars.

This question uses a very classic logic assumption – Statistical Assumption. It is similar to
hypothesis you see in Probability & Statistics of Quantitative.

The form is:


A is more likely to do X than B is
Assumption: Over the statical period, A does X more often than B does.

For example:
Team A has more chances to win this game than Team B does.
Assumption: Over the statical period, Team A wins more often than Team B

More example:
Mount A is more likely to have volcano than Mount B is
Assumption: Over the statical period, Mount A has volcano more often than Mount B

APPLY TO THIS QUESTION:

Fact: only 2 percent of drivers on Lalaland’s highways drove sports cars,


Fact: 25 percent of all vehicles ticketed for drunk driving in the past 90 days were sports cars.
Conclusion: Sports car drivers on Lalaland highways are more likely to drive drunk than are
drivers of other kinds of vehicles.
Assumption: Over the statical period, sport car drivers drives drunk more often (more
regularly) than drivers of other kind of vehicles do.

ANALYZE EACH ANSWER:


A)Drivers on Lalaland highways drive drunk more often than do drivers on highways not covered
in the report.
Wrong. Out of scope. Nothing about “drivers on highways not covered…”. We just talk about
Lalaland highways.

B)Many of the vehicles ticketed for drunk driving were ticketed more than once during the time
period covered by the report.
Wrong. TEMPTING. Why? The wording is quite simple, but if you do not read carefull, you may
assume “many of the vehicles ticked for drunk driving more than once” are “sport vehicles”.
However, B is wrong because B does not say these vehicles are sport cars. If that was the case,
B would be correct assumption. But what if the vehicles ticked for drunk more than once may
NOT be sport cars.

C)Drivers who are ticketed for drunk driving are more likely to drive drunk regularly than are
drivers who are not ticketed.
Correct. This is exactly assumption stated above.

D)The number of drivers ticketed for drunk driving was greater than the number of sportscars.
Wrong. We do not compare the number of drivers vs the number of sportcars.

E)Drivers of sports cars are less likely to be ticketed for drunk driving than are drivers of other
kinds of cars.
Wrong. It’s a reverse answer. Hence, E is wrong.

13. In the city of Cantville, a high school earns the “Young Women in Science” award for a given
year if at least ten of its female students participate in that year's state science exposition.
Last year, 80 percent of all high schools in Cantville's District 1 earned this award, while only
63 percent of all high schools in the city's District 2 earned the same award. Furthermore,
these percentages have remained at or near those respective levels throughout the entire
twelve years since the award was instituted. It is clear, then, that the high schools in District 1
do a better job of encouraging girls to pursue science than do the high schools in District 2.

Which of the following is assumed in the argument above?

(A)Students in District 1 have more free time to pursue science projects than do students in
District 2.
(B)The overall population of District 1 is not substantially larger than that of District 2.
(C)The average high school in in District 2 does not have more students than does the average
high school in District 1.
(D)District 1 does not have significantly fewer all-male high schools than does District 2.
(E)School is the primary source of encouragement for students' pursuit of science.

Frankly, all explanations above are really good. This question is NOT EASY at all. If you don't
read it carefully, you may pick right answer by using POE, but I bet you did not understand
deeply the idea that the GMAT makers want to convey in the question.

First of all, this is assumption question regarding percentage, specifically, defender assumption
(if you're interested, read Power Score CR Bible).
The most important thing here is that you must show absolute value is different from percentage
value. That's the KEY.
Let examine the question:

Premise: D1 has 80% high schools earned award


Premise: D2 has 63% high schools earned award.
Conclusion: D does better than does D2 of encouraging girls to pursue science.

We have two different cases here:

(1) DIFFERENT ABSOLUTE VALUES: D1 & D2 have different the number of schools.
(2) SAME ABSOLUTE VALUES: D1 & D2 have the same number of high school; This is the
case in the question.

For case (1): the number of schools in D1 & that of D2 are different.
The conclusion above is correct only if The number of high schools in D1 is not fewer than that of
D2. If No, the conclusion is broken.
For instance, if D1 has 100 high schools, 80% = 80 schools had award. But D2 has more, 200 high
schools for example, 63% = 126 schools had award. Yeah, we cannot say D1 did better job than
did D2. So the conclusion is not hold.
But this case is TOO OBVIOUS and normally not considered in hard GMAT question.

For case (2): the number of schools in D1 & that of D2 are the same.
Let assume: The number of high schools = the number of NON-GIRL schools (all-male) + the
number of GIRL INCLUDED schools.
So, we only can conclude D1 did better job if its number of Girl included schools is EQUAL or
SMALLER than that of D2. If NO, review the example in case 1 above.
It also mean THE NUMBER OF NON-Girl SCHOOLS (all-male) OF D1 IS NOT FEWER THAN THAT OF
D2.

That's exactly what answer D says.

14. In a fast changing market and while still single, young working professionals often change jobs
and switch job locations. Such uncertainty discourages these young professionals from buying a
home. When settled with a spouse and a stable job, however, these young professionals
generally choose to buy real estate. Surprisingly, in cities where the industry and market are
quite stable, young working professionals are still not buying, even though they are not
changing jobs and may already be settled down in a spousal relationship.

Which of the following, if true, most helps to explain the surprising finding?

(A) Stable markets do not themselves entice working professionals to buy real estate rather
than to rent it.

(B) Brokers readily connect for-sale-homes with working professionals who have stable jobs and
are in spousal relationships.

(C) The stability of industries varies widely from city to city.


(D) A very expensive home can significantly decrease the ability of a working professional to
fulfill his mortgage payments.

(E) Due to lending constraints, banks have increased the down payments required for
consumers to obtain mortgages for new homes.

ANALYZE THE STIMULUS:

Fact: Such uncertainties as fast changing market, single, and changing jobs discourage these
young professionals from buying a home.
Fact: When settled with a spouse and a stable job, these young professionals generally choose
to buy real estate.
Fact: Surprisingly, in cities where the industry and market are quite stable, young working
professionals are still not buying, even though they are not changing jobs and may already be
settled down in a spousal relationship.

Which of the following, if true, most helps to explain the surprising finding?

ANALYZE EACH ANSWER:

(A) Stable markets do not themselves entice working professionals to buy real estate rather
than to rent it.
Wrong. Sounds tempting but incorrect. “Do not entice to buy more than to rent” means
customers actually BUY real estates, but the number of homes sold do not overcome that of
home rent. Thus, A does not help to explain why customers do NOT buy real estates.

(B) Brokers readily connect for-sale-homes with working professionals who have stable jobs and
are in spousal relationships.
Wrong. Do not explain why customers do NOT buy real estates.

(C) The stability of industries varies widely from city to city.


Wrong. We do not talk about the range of variability. “Stable market” means “stable market”.
That’s it. We do not care which city has industry more stable.

(D) A very expensive home can significantly decrease the ability of a working professional to
fulfill his mortgage payments.
Wrong. We talk about real estate in general. “very expensive home” is just a sub-set of real
estates. Thus, D does not help.

(E) Due to lending constraints, banks have increased the down payments required for
consumers to obtain mortgages for new homes.
Correct. E shows the reason why consumers who have certain conditions cannot buy homes.

15. Last year, the government of country A imposed large tariffs on steel imports in an effort to aid
its domestic steel industry. Many domestic steel producers enjoyed record profits as a result,
as foreign steel producers were in many cases unable to compete effectively under the burden
of the newly imposed tariffs.

Which of the following conclusions is best supported by the passage?


A. Not all steel producers were unaffected by country A's newly imposed tariffs.
B. Some foreign steel producers were able to compete effectively in country A even after the
new tariffs were imposed.
C. After the new tariffs were imposed, most foreign steel producers were unable to compete
effectively with country A's domestic steel producers.
D. Most domestic steel producers were able to increase their profits after the new tariffs were
imposed.
E. If a government intends to protect a domestic industry, the imposition of tariffs on imports
is generally an effective approach.

Hello Ron34.

I like your comment, but your comment also demonstrates that you haven't read the question
stem carefully. If you practice hard CR questions, you will see this kind of question very often.

Okay, let see the question stem one more time.

"Which of the following conclusions is best supported by the passage?"

The difference between 600 level and 700 level lies in just one word. See the highlight word
"best". It means there are MORE THAN ONEanswers fit the conclusion. But your task is to find
the BEST one. Wow, kind of weird, right? That's how the 700 level questions are built.

GENERAL THEORIES:

Okay, back to the question. Keep in mind this is MUST BE TRUE question. You can only
deduce conclusion from premises. You CAN'T assume new information. That's the key.

ANALYZE THE STIMULUS:

Fact: Last year, the government of country A imposed large tariffs on steel imports in an effort
to aid its domestic steel industry.
Fact: Many domestic steel producers enjoyed record profits as a result, as foreign steel
producers were in many cases unable to compete effectively under the burden of the newly
imposed tariffs.

Question: Which of the following conclusions is best supported by the passage?

ANALYZE EACH ANSWER:

A. Not all steel producers were unaffected by country A's newly imposed tariffs.
Correct. Be aware of this kind of writing. DOUBLE negative = positive. So, we can rewrite A
like: SOME steel producers were AFFECTEDby country A's newly imposed tariffs. It's 100%
correct based on facts provided on the stimulus.
A should be the conclusion. Hold this one and move on.

B. Some foreign steel producers were able to compete effectively in country A even after the
new tariffs were imposed.
Wrong. Clearly, we do not know whether foreign steel producers were able to compete
effectively? We have no clues. Keep in mind this isMUST BE TRUE question, NO NEW
INFORMATION accepted. Thus, eliminate B.
C. After the new tariffs were imposed, most foreign steel producers were unable to compete
effectively with country A's domestic steel producers.
Wrong. Out at first sight. See "most". GMAT does not like extreme answer. In addition, like B,
we do not have enough clues to make up this conclusion. Eliminate C.

D. Most domestic steel producers were able to increase their profits after the new tariffs were
imposed.
Wrong. Like C, "most" is extreme word and should not be used in a correct answer.

E. If a government intends to protect a domestic industry, the imposition of tariffs on imports


is generally an effective approach.
Can be a conclusion, but not as strong as A. Because E generalize that a government can
protect all domestic industries by imposing tariffs. It maybe right, maybe wrong. We don't
know. Let narrow the scope into "steel industry" only.Thus, E can't be an answer.

Only A remains and is the best among the lot.

16. Inertia affects the flow of water pumped through a closed system of pipes. When the pump is
first switched on, the water, which has mass, takes time to reach full speed. When the pump is
switched off, inertia causes the decrease in the water flow to be gradual. The effects of
inductance in electrical circuits are similar to the effects of inertia in water pipes.

The information above provides the most support for which one of the following?

A. The rate at which electrical current flows is affected by inductance

B. The flow of electrical current in a circuit requires inertia

C. Inertia in the flow of water pumped by an electrically powered pump is caused by


inductance in the pump's circuit

D. Electrical engineers try to minimize the effects of inductance in electrical circuits

E. When a water pump is switched off it continues to pump water for a second or two

Good question. A is correct.

First of all, this is “MUST BE TRUE” question. The most important thing of this type of question
is “NO NEW information accepted”. A correct answer MUST pass the “Fact test”.
There are only 2 types of correct answer:
(1) Paraphrase (re-write parts of the stimulus in other way but keep the same meaning)
(2) Combination (combine two or more parts of the stimulus to make an inference)

ANALYZE THE STIMULUS:

Fact #1: Inertia affects the flow of water pumped through a closed system of pipes
Fact #2: The pump is on, inertia causes the mass water takes time to reach full speed.
Fact #3: The pump is off, inertia causes the decrease in the water flow to be gradual.
Fact #4: The effects of inductance in electrical circuits are similar to the effects of inertia in
water pipes.
ANALYZE EACH ANSWER:

A. The rate at which electrical current flows is affected by inductance


Correct. This is a combination of fact#1 and fact#4: flow of water is affected by inertia;
inductance affects electrical flows in the similar way as inertia does ==> Flow of electrical
current is affected by inductance.

B. The flow of electrical current in a circuit requires inertia


Wrong. “Flow of electrical current is affected by inductance, not inertia”; In addition,
“require” differs from “affects” ==> out immediately

C. Inertia in the flow of water pumped by an electrically powered pump is caused by


inductance in the pump's circuit
Wrong. No direct link between “inertia in the flow of water” and “inductance in pump’s
circuit” ==> Out immediately.

D. Electrical engineers try to minimize the effects of inductance in electrical circuits


Wrong. Nothing about “engineers try to minimize the effect of inductance” ==> Out
immediately.

E. When a water pump is switched off it continues to pump water for a second or two
Wrong. SHELL GAME. Fact #3 just says: inertia causes the decrease in the water flow to be
gradual. Thus, E is wrong because:
(1) The stimulus does not mention how long a water pump continues to pump.
(2) You cannot infer “decrease in the water flow to be gradual” = 1 or 2 seconds. It may be 3
seconds, or even a minutes, who knows?

TAKEAWAY:

In "Must be true" question:


(1) No new info accepted.
(2) A correct answer MUST pass the fact test.

17. Choi: All other factors being equal, children whose parents earned doctorates are more likely
to earn a doctorate than children whose parents did not earn doctorates.

Hart: But consider this: Over 70 percent of all doctorate holders do not have a parent that also
holds a doctorate.

Which of the following is the most accurate evaluation of Hart's reply?

(A) It establishes that Choi's claim is an exaggeration.


(B) If true, it effectively demonstrates that Choi's claim cannot be accurate.
(C) It is consistent with Choi's claim.
(D) It provides alternative reasons for accepting Choi's claim.
(E) It mistakes what is necessary for an event with what is sufficient to determine that the
event will occur.

Tough question. IMO, C is correct.


The question is: Which of the following is the most accurate evaluation of Hart's reply
So we need to find an answer that must be true for what Hart replied. Don't be simply lured
by signal words such as However, but, what more....... Make sure you understand the full
context of the argument.

Choice: Children + parents earned doctorates ==> more likely to earn a doctorate than other
children.
Hart: Over 70% of all doctorate holders do not have a parent also holds a doctorate.

Example:
There are 500 children,
30 children who have parent also hold doctorate. 20 children will earn doctorate ==>
probability = 20/30 = 67%
570 other children, only 80 children will earn doctorate ==> probability = 70/570 = 14%

Clearly, Although 80% all doctorate holders do not have a parent that also holds a doctorate.
they are less likely to earn a doctorate than children whose parents have doctorates (14% VS
67%)

==> Hart's reply is consistent with Choi.

How about other options. Why they are wrong?

(A) It establishes that Choi's claim is an exaggeration.


Wrong. Hart did not say Choi exaggerated.

(B) If true, it effectively demonstrates that Choi's claim cannot be accurate.


Wrong. Even Hart's reply is true, Choi's claim can also be true.

(C) It is consistent with Choi's claim.


Correct.

(D) It provides alternative reasons for accepting Choi's claim.


Wrong. There is no alternative reason.

(E) It mistakes what is necessary for an event with what is sufficient to determine that the
event will occur.
Wrong. There are not necessary condition and sufficient condition in the argument. Moreover,
Hart's reply is actually not a sufficient condition for Choi's claim.

Das könnte Ihnen auch gefallen